Wekening - Govt. Plan

This topic has expert replies
Senior | Next Rank: 100 Posts
Posts: 52
Joined: Tue Apr 14, 2009 1:23 am

Wekening - Govt. Plan

by Gmat09_5ALL » Wed Sep 30, 2009 9:28 pm
6.20
Five years ago, as part of a plan to encourage citizens of Levaska to increase the amount of money they put into savings, Levaska's government introduced special savings accounts in which up to $3,000 a year can be saved with no tax due on the interest unless money is withdrawn before the account holder reaches the age of sixty-five. Millions of dollars have accumulated in the special accounts, so the government's plan is obviously working.
Which of the following, if true, most seriously weakens the argument?
A. A substantial number of Levaskans have withdrawn at least some of the money they had invested in the special accounts.
B. Workers in Levaska who already save money in long-term tax-free accounts that are offered through their workplace cannot take advantage of the special savings accounts introduced by the government.
C. The rate at which interest earned on money deposited in regular savings accounts is taxed depends on the income bracket of the account holder.
D. Many Levaskans who already had long-term savings have steadily been transferring those savings into the special accounts.
E. Many of the economists who now claim that the government's plan has been successful criticized it when it was introduced.

Master | Next Rank: 500 Posts
Posts: 239
Joined: Wed Feb 11, 2009 2:50 am

by delhiboy1979 » Thu Oct 01, 2009 12:15 am
I think it should be D.

D implies that there is no additional savings. THe citizens already had the money in the savings and habve onnly been moving them across accounts. So logically, there is no extra savings.

Legendary Member
Posts: 1404
Joined: Tue May 20, 2008 6:55 pm
Thanked: 18 times
Followed by:2 members

by tanviet » Thu Oct 01, 2009 6:39 am
Pls, help, why A is wrong

User avatar
Master | Next Rank: 500 Posts
Posts: 106
Joined: Mon Sep 28, 2009 9:29 am
Location: Boston, MA
Thanked: 26 times
Followed by:2 members
GMAT Score:700

by asamaverick » Thu Oct 01, 2009 8:20 am
IMO A is wrong because -

The government's objective was to increase the money in savings and from the concluding line it is clear that a lot of money did go into this special savings account.

As per 'A', substantial number of people withdrew at least some money. This in no way indicates that people did not start saving because of the government initiative. Hence it is not enough to state (based on A) that government failed in its objective.

Legendary Member
Posts: 1404
Joined: Tue May 20, 2008 6:55 pm
Thanked: 18 times
Followed by:2 members

why A is wrong, very hard CR

by tanviet » Fri Oct 02, 2009 12:53 am
I still do not understand why A is wrong. pls help

Senior | Next Rank: 100 Posts
Posts: 76
Joined: Fri Dec 26, 2008 8:18 am
Thanked: 3 times

by srivas » Sat Oct 03, 2009 5:28 am
Millions of dollars have accumulated in the special accounts, so the government’s plan is obviously working

In option D people are trasforing money from the exisiting accounts to the special accounts.
Gmat710,, Hyd

Master | Next Rank: 500 Posts
Posts: 280
Joined: Tue Sep 30, 2008 4:18 am
Thanked: 5 times
GMAT Score:610

Re: Wekening - Govt. Plan

by Jatinder » Sun Oct 04, 2009 4:53 am
Gmat09_5ALL wrote:6.20
Five years ago, as part of a plan to encourage citizens of Levaska to increase the amount of money they put into savings, Levaska's government introduced special savings accounts in which up to $3,000 a year can be saved with no tax due on the interest unless money is withdrawn before the account holder reaches the age of sixty-five. Millions of dollars have accumulated in the special accounts, so the government's plan is obviously working.
Which of the following, if true, most seriously weakens the argument?
A. A substantial number of Levaskans have withdrawn at least some of the money they had invested in the special accounts.
B. Workers in Levaska who already save money in long-term tax-free accounts that are offered through their workplace cannot take advantage of the special savings accounts introduced by the government.
C. The rate at which interest earned on money deposited in regular savings accounts is taxed depends on the income bracket of the account holder.
D. Many Levaskans who already had long-term savings have steadily been transferring those savings into the special accounts.
E. Many of the economists who now claim that the government's plan has been successful criticized it when it was introduced.
What is the source of the question Gmat09?
Keep flying

Master | Next Rank: 500 Posts
Posts: 295
Joined: Tue Jul 15, 2008 10:07 am
Thanked: 4 times
GMAT Score:690

by vaibhav.iit2002 » Mon Oct 05, 2009 6:58 am
Should be D

Master | Next Rank: 500 Posts
Posts: 153
Joined: Tue Jun 02, 2009 2:48 pm
Location: Ohio
Thanked: 16 times

by JeffB » Mon Oct 05, 2009 4:11 pm
D strengthens the argument, does not weaken it.

Newbie | Next Rank: 10 Posts
Posts: 1
Joined: Sat Dec 06, 2008 11:55 am
Location: Bangalore

by nawabkhan83 » Tue Oct 06, 2009 3:21 am
JeffB wrote:D strengthens the argument, does not weaken it.
I think it should be either C or D.
D looks a good option

Legendary Member
Posts: 1404
Joined: Tue May 20, 2008 6:55 pm
Thanked: 18 times
Followed by:2 members

by tanviet » Thu Oct 15, 2009 1:20 am
Pls, help explain, why A is wrong

Senior | Next Rank: 100 Posts
Posts: 79
Joined: Sat May 24, 2008 8:57 pm
Location: O-Town FL
Thanked: 1 times

by LifetimesofSC » Thu Oct 15, 2009 8:51 am
I fell for the trap but I see it now...and obviously its too late.

1.) Govt wants citizens to increase the $ they put into savings.

2.) A is tricky as it states 'substantial', however D states that citizens have been transferring money from 1 savings account to another, meaning they are not increasing their savings but only balancing/allocating it.

F*** the GMAC/ETS for creating this question

Legendary Member
Posts: 2326
Joined: Mon Jul 28, 2008 3:54 am
Thanked: 173 times
Followed by:2 members
GMAT Score:710

Re: Wekening - Govt. Plan

by gmatmachoman » Thu Oct 15, 2009 10:26 pm
Gmat09_5ALL wrote:6.20
Five years ago, as part of a plan to encourage citizens of Levaska to increase the amount of money they put into savings, Levaska's government introduced special savings accounts in which up to $3,000 a year can be saved with no tax due on the interest unless money is withdrawn before the account holder reaches the age of sixty-five. Millions of dollars have accumulated in the special accounts, so the government's plan is obviously working.
Which of the following, if true, most seriously weakens the argument?
A. A substantial number of Levaskans have withdrawn at least some of the money they had invested in the special accounts.
B. Workers in Levaska who already save money in long-term tax-free accounts that are offered through their workplace cannot take advantage of the special savings accounts introduced by the government.
C. The rate at which interest earned on money deposited in regular savings accounts is taxed depends on the income bracket of the account holder.
D. Many Levaskans who already had long-term savings have steadily been transferring those savings into the special accounts.
E. Many of the economists who now claim that the government's plan has been successful criticized it when it was introduced.

Is it not B???

Legendary Member
Posts: 1404
Joined: Tue May 20, 2008 6:55 pm
Thanked: 18 times
Followed by:2 members

by tanviet » Thu Oct 15, 2009 11:23 pm
Pls, explain why A is wrong. A said that a number of people take out money. this is weakening.

pls, help me out.

Legendary Member
Posts: 1404
Joined: Tue May 20, 2008 6:55 pm
Thanked: 18 times
Followed by:2 members

pls,help me, why A is wrong, weakening

by tanviet » Thu Oct 15, 2009 11:42 pm
pls, help me why A is wrong